LSAT and Law School Admissions Forum

Get expert LSAT preparation and law school admissions advice from PowerScore Test Preparation.

User avatar
 Dave Killoran
PowerScore Staff
  • PowerScore Staff
  • Posts: 5972
  • Joined: Mar 25, 2011
|
#22748
Complete Question Explanation

Flaw in the Reasoning-SN. The correct answer choice is (B)

The stimulus contains a Mistaken Reversal:
  • SS = switch suppliers ..... P = profit

    Premise: SS → P

    Premise: P

    Conclusion: SS
According to the premise, switching suppliers is the sufficient condition ("switching suppliers") under which a certain phenomenon ("no profit") is said to occur. But, just because Blankenship shows no profit for the year, does that automatically mean that they switched suppliers? No, they could have had a down year or something else negative could have occurred. But, the author clearly assumes that a no profit year means a switch of suppliers, which is a Mistaken Reversal of the first premise. The task now becomes finding the answer that best describes the Reversal.

As far as this question, focus on two things here:

  • 1. This argument contains a flaw.

    2. The flaw is a Mistaken Reversal, which occurs when an author mixes up conditional terms by reversing them.

    3. Knowing that this is a Mistaken Reversal, you should seek a description of that in the answer choices.

Answer choice (A): This answer describes a Circular Reasoning flaw, which is mostly used as a wrong answer (but occasionally as a correct answer), and circular arguments have premises and conclusions that are roughly identical, or have an argument form that assumes the conclusion is true. In this case, the second claim in the stimulus is clearly not the same as the first claim, and thus this is not a circular argument (a Mistaken Reversal is not the same as circular reasoning).

Answer choice (B): This description is difficult to understand, but note how it is the only answer to refer to "conditions"—this is a tipoff that you should examine this answer closely given the conditional nature of the flaw in this problem. In this case, "condition" refers to "switching suppliers," whereas the "phenomenon" refers to the lack of profit. Since switching suppliers is not necessarily the only condition under which the company would show no profit, answer choice (B) properly describes the Mistaken Reversal. Note: Answer choice (B) is a classic description of a Mistaken Reversal, so take a moment to make sure you understand the form of the description.

Answer choice (C): This answer is incorrect since the stimulus does not use a term or phrase in an equivocal or uncertain manner.

Answer choice (D): This answer is incorrect because the stimulus does not use an "exceptional, isolated case" to support its conclusion.

Answer choice (E): This answer is incorrect because it describes causal reasoning, and we know that in this case the error in the stimulus involves conditional reasoning.
 ellenb
  • Posts: 260
  • Joined: Oct 22, 2012
|
#11831
Dear Powerscore,

I got this answer correct, however, I want to make sure I understand correctly what the answer is saying when it says "the argument fails to establish that a condition under which a phenomenon is said to occur is the only condition under which that phenomenon occurs"

Thus, is it based on the second statement or based or based on the first statement.

I thought that the condition is the first part and phenomenon is the second part.
Thus we would have A-->not B, and if I had not B--> A, which is a mistaken reversal, so basically it is saying that not B (condition)? and A (phenomenon)?
so how would the answer choice apply to this example, and what does it actually mean.

Thanks in advance!

Ellen
 Steve Stein
PowerScore Staff
  • PowerScore Staff
  • Posts: 1153
  • Joined: Apr 11, 2011
|
#11833
Hi Ellen,

The part of the answer choice that you quoted basically says that the author doesn't establish that the phenomenon discussed will happen only under the given condition.

...i.e., the author does not establish that: Phenomenon :arrow: Condition

More specifically, in this case the author fails to establish that a profitless year could only happen in the case of a supplier-switch.

...i.e., the author does not establish that: Profitless year :arrow: Supplier-switch

I hope that's helpful! Please let me know whether this is clear--thanks!

~Steve
 ellenb
  • Posts: 260
  • Joined: Oct 22, 2012
|
#11873
Thanks Steve, in other words, a mistaken Reversal Flaw, could be said to be the "the author does not establish that the phenomenon discussed will happen only under the given condition"
A-->B, B-->A, (the phenomenon A happened in the second example when b was the condition).


Please let me know if I understood you correctly.

Thanks

Ellen
 JennuineInc
  • Posts: 18
  • Joined: May 11, 2016
|
#24472
Hi,

I got B through process of elimination but I still don't understand exactly what the correct answer B means in normal english using the stimulus.

The answer choice says:
B. "Fails to establish that a condition under which a phenomenon is said to occur is the only condition under which that phenomenon occurs"

"The Condition" being ~P = Not Show a Profit

"The Phenomenon being SS = Switch suppliers

Isn't the answer saying the argument fails to establish that the necessary condition is the only condition which the sufficient condition occurs:
The argument fails to establish that the condition of not showing a profit under which a phenomenon of switching suppliers is said to occur, is the only condition of not showing a profit which the phenomenon of switching suppliers occurs.

Why would the answer say the argument FAILS to establish a mistake reversal? It doesn't say fails to establish the TRUTH of the mistake reversal.
But isn't that what the author is doing, establishing a mistake reversal?
I think I've confused myself. Can someone please explain what the answer B is actually saying?
 Nikki Siclunov
PowerScore Staff
  • PowerScore Staff
  • Posts: 1362
  • Joined: Aug 02, 2011
|
#24593
Hi JennuineInc,

Thanks for your question. Answer choice (B) is indeed worded in a convoluted way. But, let's take a step back and analyze what a Mistaken Reversal is, and why it's a flaw:

In the simplest terms possible, a Mistaken Reversal is an erroneous inference that confuses a necessary condition for a sufficient condition. (Law schools admit only those who take the LSAT. Mary took the LSAT, therefore she will gain admission to at least one law school). This concept can be introduced (and described) in many ways, but the core message is the same: the author assumes, erroneously, that a condition necessary for a certain event to occur (the LSAT is necessary for admission) is actually sufficient for that event to occur (Mary's taking the LSAT is enough to ensure that she will get in).

Now, let's take a look at the question at hand: according to the premise, switching suppliers is the condition under which a certain phenomenon ("no profit") is said to occur. But, is this the only condition under which that phenomenon can ever occur? In other words, does Blankenship show no profit only when switching suppliers??). The author clearly assumes so, which is a Mistaken Reversal of the premise. Your understanding of the answer choice (B) is incorrect, because the "condition" refers to "switching suppliers," whereas the "phenomenon" refers to the lack of profit. Since switching suppliers is not necessarily the only condition under which the company would show no profit, answer choice (B) properly describes the Mistaken Reversal at play.

Hope this clears things up! :-)
 JennuineInc
  • Posts: 18
  • Joined: May 11, 2016
|
#25114
Thank you Nikki! Your explanation was great! :-D

So essentially the answer saying if the author HAD established that as a premise, then the mistake reversal that the author made WOULD HAVE been correct. But in failing to establish that, the author made a mistake reversal.

I see that happen in Flaw questions often. Where sometimes the question stem asks what the author fails to establish. Is it usually asking for the missing premise that would have made the conclusion correct?

Thanks so much!!!
 Jon Denning
PowerScore Staff
  • PowerScore Staff
  • Posts: 907
  • Joined: Apr 11, 2011
|
#25303
Hey Jenuine,

Usually if you see a missing piece like that it's the key to the question, whether in Flaw (like, "The author fails to establish...") or Assumption or Justify or Weaken/Strengthen, etc. I stress to my students all the time the notion of logical "gaps" or "leaps," where an author moves from one idea to a new one without explicitly bridging the space between, as recognizing those moments is of central importance to LR success.

So I'm pleased to see you focusing on it! Keep it up!
 jwrig9
  • Posts: 2
  • Joined: Aug 21, 2017
|
#38863
I understand this is a conditional reasoning flaw, however I do not understand how it is not also considered circular. Could you please give me an example of what a circular flaw would look like? This is what I expected but I not see it is not...

Get the most out of your LSAT Prep Plus subscription.

Analyze and track your performance with our Testing and Analytics Package.